Paleontologist: Plesiosauromorphs were gigantic, long-necked marine reptiles that ruled the oceans during the age of ...

Missy on November 2, 2015

Explanation

Can you please explain why E is the correct answer

Replies
Create a free account to read and take part in forum discussions.

Already have an account? log in

Mehran on November 17, 2015

Thank you for your question. This strengthening question asks that we identify an assumption on which the argument in the stimulus depends. Let's first be sure we understand each part of the stimulus itself.

The stimulus is an argument. The conclusion is that "plesiosauromorphs probably hunted by chasing their prey over long distances." The only premise offered in support of this conclusion is that these creatures had fins that were "quite long and thin, like the wings of birds specialized for long-distance flight."

Let's look at this closely. Notice how this premise is really drawing an analogy of some kind--that something relevant to birds (specifically, the shape of a bird's wings) can tell us something about the hunting habits of plesiosauromorphs (specifically, the kinds of distances these creatures covered while hunting).

Well, there's a gap there--an assumption. The unstated premise is set out in answer choice (E): "the shape of a marine animal's fin affects the way the animal swims in the same way as the shape of a bird's wing affects the way the bird flies."

Hope this helps! Please let us know if you have any additional questions.

Shiyi on February 24, 2019

Why is C incorrect?

Ravi on February 26, 2019

@Shiyi-Zhang,

Happy to help.

(C) says, "A gigantic marine animal would not be able to find enough
food to meet the caloric requirements dictated by its body size if it
did not hunt by chasing prey over long distances."

The negation of (C) is that a gigantic marine animal would be able to
find enough food to meet the caloric requirements dictated by its body
size if it did not hunt by chasing prey over long distances.

The negation of (C) does not wreck the argument. Looking at (C), it's
a very strong answer choice. If this question were a strengthen with a
SUFFICIENT premise, then this would be the correct answer. However,
we're looking for the answer that the argument NEEDS, and (C)'s
wording is too strong. (C) simply doesn't have to be true in order for
the argument to hold. While it would make the argument valid, we don't
need it, so we can get rid of this answer choice.

Does this make sense? Let us know if you have any other questions!